www.vorhilfe.de
Vorhilfe

Kostenlose Kommunikationsplattform für gegenseitige Hilfestellungen.
Hallo Gast!einloggen | registrieren ]
Startseite · Forum · Wissen · Kurse · Mitglieder · Team · Impressum
Forenbaum
^ Forenbaum
Status Englisch
  Status Grammatik
  Status Lektüre
  Status Korrekturlesen
  Status Übersetzung
  Status Sonstiges (Englisch)

Gezeigt werden alle Foren bis zur Tiefe 2

Navigation
 Startseite...
 Neuerdings beta neu
 Forum...
 vorwissen...
 vorkurse...
 Werkzeuge...
 Nachhilfevermittlung beta...
 Online-Spiele beta
 Suchen
 Verein...
 Impressum
Das Projekt
Server und Internetanbindung werden durch Spenden finanziert.
Organisiert wird das Projekt von unserem Koordinatorenteam.
Hunderte Mitglieder helfen ehrenamtlich in unseren moderierten Foren.
Anbieter der Seite ist der gemeinnützige Verein "Vorhilfe.de e.V.".
Partnerseiten
Weitere Fächer:

Open Source FunktionenplotterFunkyPlot: Kostenloser und quelloffener Funktionenplotter für Linux und andere Betriebssysteme
Forum "Reelle Analysis mehrerer Veränderlichen" - Extrema mit Lagrange
Extrema mit Lagrange < mehrere Veränderl. < reell < Analysis < Hochschule < Mathe < Vorhilfe
Ansicht: [ geschachtelt ] | ^ Forum "Reelle Analysis mehrerer Veränderlichen"  | ^^ Alle Foren  | ^ Forenbaum  | Materialien

Extrema mit Lagrange: Frage (beantwortet)
Status: (Frage) beantwortet Status 
Datum: 11:47 So 06.07.2014
Autor: SturmGhost

Aufgabe
Bestimmen Sie mithilfe der Methode von Lagrange das globale Minimum der Zielfunktion f(x,y)=2x+2y, dessen Existenz vorausgesetzt werden darf, unter der Nebenbedingung [mm] g(x,y)=x^2+y^2=2 [/mm]

Bitte mal drüberschauen ob ich das richtig gemacht habe:

Also zunächst muss gelten

[mm] grad(g(x,y))\not=0 [/mm]
grad(g(x,y))= (2x , 2y)

Da 0 zu einem Widerspruch in der NB führt muss gelten [mm] x,y\in\IR\backslash\{0\} [/mm]

Also nun Lagrange:

[mm] L(x,y,\lambda)=2x+2y+\lambda*(x^2+y^2-2) [/mm]

[mm] grad(L(x,y,\lambda))=(2+\lambda2x [/mm] , [mm] 2+\lambda2y [/mm] , [mm] x^2+y^2-2)=0 [/mm]

NLGS:

I.  [mm] 2+\lambda2x=0 [/mm]
II. [mm] 2+\lambda2y=0 [/mm]
III. [mm] x^2+y^2-2=0 [/mm]

Also:

I.  [mm] 2y+\lambda2xy=0 [/mm]
II. [mm] 2x+\lambda2xy=0 [/mm]
III. [mm] x^2+y^2-2=0 [/mm]

Also:

2y=2x [mm] \Rightarrow [/mm] x=y

Nun:

[mm] x^2+y^2-2=0 \Rightarrow x^2+x^2-2=0 \gdw 2x^2-2=0 \gdw x=\pm1 [/mm]

Und somit:

[mm] y=\pm1 [/mm]

Also P1 (1,1) und P2 (-1,-1)

Schlussendlich:

f(1,1)= 4 und f(-1,-1)= -4

Also liegt bei P1 (1,1) ein globales Minimum vor da f(1,1)>0?

        
Bezug
Extrema mit Lagrange: Antwort
Status: (Antwort) fertig Status 
Datum: 11:57 So 06.07.2014
Autor: fred97


> Bestimmen Sie mithilfe der Methode von Lagrange das globale
> Minimum der Zielfunktion f(x,y)=2x+2y, dessen Existenz
> vorausgesetzt werden darf, unter der Nebenbedingung
> [mm]g(x,y)=x^2+y^2=2[/mm]
>  Bitte mal drüberschauen ob ich das richtig gemacht habe:
>  
> Also zunächst muss gelten
>
> [mm]grad(g(x,y))\not=0[/mm]
>  grad(g(x,y))= (2x , 2y)
>  
> Da 0 zu einem Widerspruch in der NB führt muss gelten
> [mm]x,y\in\IR\backslash\{0\}[/mm]
>  
> Also nun Lagrange:
>  
> [mm]L(x,y,\lambda)=2x+2y+\lambda*(x^2+y^2-2)[/mm]
>  
> [mm]grad(L(x,y,\lambda))=(2+\lambda2x[/mm] , [mm]2+\lambda2y[/mm] ,
> [mm]x^2+y^2-2)=0[/mm]
>
> NLGS:
>  
> I.  [mm]2+\lambda2x=0[/mm]
>  II. [mm]2+\lambda2y=0[/mm]
>  III. [mm]x^2+y^2-2=0[/mm]
>  
> Also:
>  
> I.  [mm]2y+\lambda2xy=0[/mm]
>  II. [mm]2x+\lambda2xy=0[/mm]
>  III. [mm]x^2+y^2-2=0[/mm]
>  
> Also:
>  
> 2y=2x [mm]\Rightarrow[/mm] x=y
>  
> Nun:
>
> [mm]x^2+y^2-2=0 \Rightarrow x^2+x^2-2=0 \gdw 2x^2-2=0 \gdw x=\pm1[/mm]
>  
> Und somit:
>  
> [mm]y=\pm1[/mm]
>  
> Also P1 (1,1) und P2 (-1,-1)
>  
> Schlussendlich:
>  
> f(1,1)= 4 und f(-1,-1)= -4
>
> Also liegt bei P1 (1,1) ein globales Minimum vor da
> f(1,1)>0?  


Nein. f nimmt sein in (-1,-1) an.

FRED

Bezug
                
Bezug
Extrema mit Lagrange: Frage (beantwortet)
Status: (Frage) beantwortet Status 
Datum: 15:18 So 06.07.2014
Autor: SturmGhost

Wie kommt man darauf?

Bezug
                        
Bezug
Extrema mit Lagrange: Antwort
Status: (Antwort) fertig Status 
Datum: 15:56 So 06.07.2014
Autor: leduart

Hallo
du hast Ectremwerte in (1,1) und (-1,-1) aber der Funktionswert bei (-1,-1) ist kleiner als bei (1,1) wie du ja geschrieben hast, deshalb kann ja bei (1,1) kein Minimum sein!
(du verwechselst da was mit f'' >0  bei  Minima von f(x)
Gruss leduart

Bezug
Ansicht: [ geschachtelt ] | ^ Forum "Reelle Analysis mehrerer Veränderlichen"  | ^^ Alle Foren  | ^ Forenbaum  | Materialien


^ Seitenanfang ^
www.englischraum.de
[ Startseite | Forum | Wissen | Kurse | Mitglieder | Team | Impressum ]